Đến nội dung

Phạm Hữu Bảo Chung nội dung

Có 549 mục bởi Phạm Hữu Bảo Chung (Tìm giới hạn từ 04-06-2020)



Sắp theo                Sắp xếp  

#335390 CMR: $c+d>a+b$

Đã gửi bởi Phạm Hữu Bảo Chung on 13-07-2012 - 21:58 trong Bất đẳng thức và cực trị

Cho $a$, $b$, $c$ là các số thực dương. Chứng minh rằng trong các bất đẳng thức sau có ít nhất một bất đẳng thức sai:
$c+d>a+b$, $ab+cd>(a+b)(c+d)$, $ab(c+d)>(a+b)cd$

Giải

Giả sử cả 3 BĐT đều đúng.
Ta có $(3) \Leftrightarrow ac(b - d) + bd(a - c) > 0$
Dễ thấy:
- Nếu b < d và a < c thì BPT nói trên sai.

- Nếu b > d và a > c thì suy ra $a + b > c + d$. Điều này mâu thuẫn với (1)
Suy ra, $\left[\begin{array}{l} \left\{\begin{array}{l}b < d\\a > c\end{array}\right.\\\left\{\begin{array}{l}b > d\\a < c\end{array}\right.\end{array}\right. \Rightarrow (b - d)(a - c) < 0$
$$\Rightarrow ab + cd < bc + ad$$

Khi đó, (2) tương đương:
$$bc + ad > ab + cd > ac + ad + bc + bd \Leftrightarrow ac + bd < 0$$

Với a, b, c dương thì điều này là vô lý. Vậy có ít nhất một bất đẳng thức sai.



#335326 Giải các pt, hệ pt: 2.$\sqrt{2x^{2}+9}+\sq...

Đã gửi bởi Phạm Hữu Bảo Chung on 13-07-2012 - 20:19 trong Phương trình - hệ phương trình - bất phương trình

Giải phương trình:
$\sqrt{x + 1} + 6\sqrt{9 - x^2} + 6\sqrt{(x + 1)(9 - x^2)} = 38 + 10x - 2x^2 - x^3$

Giải

ĐK:
$\left\{\begin{array}{l}x \geq - 1\\9 - x^2 \geq 0\end{array}\right. \Leftrightarrow \left\{\begin{array}{l}x \geq -1\\-3 \leq x \leq 3\end{array}\right.$

$\Leftrightarrow -1 \leq x \leq 3$


Đặt $\left\{\begin{array}{l}a = \sqrt{x + 1} \geq 0\\b = \sqrt{9 - x^2} \geq 0\end{array}\right.$

Ta sẽ biểu diễn VF phương trình ban đầu theo a và b. Ta có:
$38 + 10x - 2x^2 - x^3 = (9x - x^3 + 9 - x^2) + 9 - x^2 + x + 1 + 19$

$= (9 - x^2)(x + 1) + (9 - x^2) + (x + 1) + 19 = a^2b^2 + b^2 + a^2 + 19$


Từ đó suy ra, PT ban đầu tương đương:
$a + 6b + 6ab = a^2b^2 + b^2 + a^2 + 19$

$\Leftrightarrow (a^2b^2 - 6ab + 9) + (a^2 - a + \dfrac{1}{4}) + (b^2 - 6b + 9) + \dfrac{3}{4} = 0$

$\Leftrightarrow (ab - 3)^2 + (a - \dfrac{1}{2})^2 + (b - 3)^2 + \dfrac{3}{4} = 0$

Phương trình nói trên vô nghiệm. Vậy phương trình ban đầu vô nghiệm.



#335027 Giải hệ: $a^{2}=\frac{\sqrt{bc}\...

Đã gửi bởi Phạm Hữu Bảo Chung on 12-07-2012 - 22:37 trong Phương trình - hệ phương trình - bất phương trình

$\left\{\begin{array}{l}a^2 = \dfrac{\sqrt{bc}.\sqrt[3]{bcd}}{(b + c)(b + c + d)}\\b^2 = \dfrac{\sqrt{cd}.\sqrt[3]{cda}}{(c + d)(c + d + a)}\\c^2 = \dfrac{\sqrt{da}.\sqrt[3]{dab}}{(d + a)(d + a + b)}\\d^2 = \dfrac{\sqrt{ab}.\sqrt[3]{abc}}{(a + b)(a + b + c)}\end{array}\right.$

Giải

ĐK:
$\left\{\begin{array}{l}bc \geq 0\\cd \geq 0\\da \geq 0\\ab \geq 0\\(b + c)(b + c + d) \neq 0\\(c + d)(c + d + a) \neq 0\\(d + a)(d + a + b) \neq 0\\(a + b)(a + b + c) \neq 0\end{array}\right. $


- Nếu $a = 0 \Rightarrow d^2 = 0 \Leftrightarrow d = 0$
Vậy $a + d = 0$. Không thỏa mãn điều kiện đề bài.

Vậy $a \neq 0$. Tương tự $b, c, d \neq 0$

$\Leftrightarrow \left[\begin{array}{l} \left\{\begin{array}{l}a > 0\\b > 0\\c > 0\\d > 0\end{array}\right.\\\left\{\begin{array}{l}a < 0\\b < 0\\c < 0\\d < 0\end{array}\right.\end{array}\right.$

Nhận thấy nếu cả 4 số đều âm thì: $a^2 = \dfrac{\sqrt{bc}.\sqrt[3]{bcd}}{(b + c)(b + c + d)} < 0$. Vô lý. Từ đó suy ra cả 4 số đó đều dương.

- Từ phương trình thứ nhất của hệ, ta có:
$a^2 = \dfrac{\sqrt{bc}.\sqrt[3]{bcd}}{(b + c)(b + c + d)} \leq \dfrac{\dfrac{b + c}{2}.\dfrac{b + c + d}{3}}{(b + c)(b + c + d)} = \dfrac{1}{6} $

Tương tự, ta cũng có: $b^2, c^2, d^2 \leq \dfrac{1}{6}$ kết hợp với $a, b, c > 0$. Suy ra:
$$0 < a, b, c, d \leq \dfrac{1}{\sqrt{6}} \Rightarrow S = a + b + c + d \leq \dfrac{4}{\sqrt{6}}$$


Nhân cả vế theo vế của 4 phương trình, ta được:
$a^2b^2c^2d^2 = \dfrac{\sqrt{a^2b^2c^2d^2}.\sqrt[3]{a^3b^3c^3d^3}}{(b + c)(b + c + d)(c + d)(c + d + a)(d + a)(d + a + b)(a + b)(a + b + c)}$

$\Leftrightarrow 1 = \dfrac{1}{(b + c)(b + c + d)(c + d)(c + d + a)(d + a)(d + a + b)(a + b)(a + b + c)}$

$\Leftrightarrow (b + c)(b + c + d)(c + d)(c + d + a)(d + a)(d + a + b)(a + b)(a + b + c) = 1$


Ta thấy:
$(b + c)(b + c + d)(c + d)(c + d + a)(d + a)(d + a + b)(a + b)(a + b + c)$


$= [(b + c)(c + d)(d + a)(a + b)][(b + c + d)(c + d + a)(d + a + b)(a + b + c)]$

$\leq \dfrac{(2S)^4}{4^4}.\dfrac{(3S)^4}{4^4} = \dfrac{2^4.3^4.S^8}{2^{16}} \leq \dfrac{2^4.3^4.\dfrac{2^{16}}{2^4.3^4}}{2^{16}} = 1 = VF$


Dấu "=" xảy ra khi: $a = b = c = d = \dfrac{1}{\sqrt{6}}$

Vậy hệ có nghiệm $(a; b; c; d) = (\dfrac{1}{\sqrt{6}}; \dfrac{1}{\sqrt{6}}; \dfrac{1}{\sqrt{6}}; \dfrac{1}{\sqrt{6}})$



#334347 Giải PT : $(8sin^{3}x+1)^{3}-162 sinx +27= 0$

Đã gửi bởi Phạm Hữu Bảo Chung on 11-07-2012 - 11:59 trong Phương trình, Hệ phương trình Lượng giác

Em chưa học Toán 11 nên chưa học phần arc. Nhưng em nghĩ là $\tan{x} = \pm 1 \Rightarrow \sin{x} = \pm \cos{x}$
Thế vào $\cos^2{x} + \sin^2{x} = 1$ thì ta được các nghiệm nói trên :)




#333985 Giải PT : $(8sin^{3}x+1)^{3}-162 sinx +27= 0$

Đã gửi bởi Phạm Hữu Bảo Chung on 10-07-2012 - 13:48 trong Phương trình, Hệ phương trình Lượng giác

Bài 2:
$3(1+\frac{\cos{2x}}{\cos^2{x}})^4+ 4\tan^6{x} = 7$

Giải

Phương trình ban đầu tương đương:
$3(1 + \dfrac{\cos^2{x} - \sin^2{x}}{\cos^2{x}})^4 + 4\tan^6{x} = 7$

$\Leftrightarrow 3(2 - \tan^2{x})^4 + 4\tan^6{x} = 7 \,\, (2)$

Đặt $a = \tan^2{x} \geq 0$. Phương trình (2) trở thành:
$3(2 - a)^4 + 4a^3 = 7 \Leftrightarrow 3a^4 - 20a^3 + 72a^2 - 96a + 41 = 0$

$\Leftrightarrow (a - 1)^2(3a^2 - 14a + 41) = 0$

$\Rightarrow \left[\begin{array}{l} a = 1\\3a^2 - 14a + 41 = 0 \,\, (VN)\end{array}\right.$


$\Rightarrow \tan^2{x} = 1 \Leftrightarrow \tan{x} = \pm 1$

$\Rightarrow \left[\begin{array}{l} x = \dfrac{\pm \pi}{4} + 2k\pi\\x = \dfrac{\pm 3\pi}{4} + 2k\pi\end{array}\right.$



#333862 Phương trình lượng giác: $(1+sin^{2}x)cosx+(1+cos^{2}x)sinx=1+sin2x$

Đã gửi bởi Phạm Hữu Bảo Chung on 09-07-2012 - 23:12 trong Phương trình, Hệ phương trình Lượng giác

Giải phương trình:
$$(1+\sin^2{x})\cos{x}+(1+\cos^2{x})\sin{x}=1+\sin{2x}$$

Giải

Phương trình tương đương:
$\sin{x} + \cos{x} + \sin^2{x}.\cos{x} + \cos^2{x}.\sin{x} = \sin^2{x} +\cos^2{x} + 2\sin{x}.\cos{x}$


$\Leftrightarrow (\sin{x} + \cos{x})( 1 + \sin{x}.\cos{x}) = (\sin{x} + \cos{x})^2$

$\Leftrightarrow (\sin{x} + \cos{x})(\sin{x}.\cos{x} - \sin{x} - \cos{x} - 1) = 0$

$\Leftrightarrow (\sin{x} + \cos{x})(\sin{x} - 1)(\cos{x} - 1) = 0$

$\Rightarrow \left[\begin{array}{l} \sin{x} = -\cos{x}\\\sin{x} = 1\\\cos{x} = 1\end{array}\right. \Rightarrow \left[\begin{array}{l} x = \dfrac{3\pi}{4} + k\pi\\x = \dfrac{\pi}{2} + 2k\pi\\x = 2k\pi\end{array}\right. \,\, (k \in Z)$



#333856 Phương trình lượng giác: $sin2x-cos2x+3sinx-cosx-1=0$

Đã gửi bởi Phạm Hữu Bảo Chung on 09-07-2012 - 23:00 trong Phương trình, Hệ phương trình Lượng giác

Giải phương trình:
$$\sin{2x} - \cos{2x} + 3\sin{x} - \cos{x} - 1 = 0$$

Giải

Phương trình tương đương:
$2\sin{x}.\cos{x} - 1 + 2\sin^2{x} + 3\sin{x} - \cos{x} - 1 = 0$


$\Leftrightarrow \cos{x}(2\sin{x} - 1) + \sin{x}(2\sin{x} - 1) + 2(2\sin{x} - 1) = 0$

$\Leftrightarrow (2\sin{x} - 1)(\cos{x} + \sin{x} + 2) = 0$


$\Leftrightarrow \left[\begin{array}{l} \sin{x} = \dfrac{1}{2}\\\sin{x} + \cos{x} = -2\end{array}\right. \,\, (1)$

Ta thấy: $\sin{x} + \cos{x} = \sqrt{2}.\sin{(x + \dfrac{\pi}{4})} \in [- \sqrt{2}; \sqrt{2}] \Rightarrow \sin{x} + \cos{x} > -2$

Do đó:
$(1) \Leftrightarrow \sin{x} = \dfrac{1}{2} \Rightarrow \left[\begin{array}{l} \dfrac{\pi}{6} + 2k\pi\\x = \dfrac{5\pi}{6} + 2k\pi\end{array}\right. \,\, (k \in Z)$



#333851 pt lượng giác: $sin2xcosx+sinxcosx=cos2x+sinx+cosx$

Đã gửi bởi Phạm Hữu Bảo Chung on 09-07-2012 - 22:47 trong Phương trình, Hệ phương trình Lượng giác

Giải phương trình:
$\sin{2x} \cos{x} + \sin{x} \cos{x} = \cos{2x} + \sin{x} + \cos{x}$

Giải

Phương trình tương đương:
$2.\sin{x}.\cos^2{x} + \sin{x}.\cos{x} - \sin{x} - \cos{x} + 1 - 2\cos^2{{x}} = 0$


$\Leftrightarrow 2\cos^2{x}(\sin{x} - 1) + (\sin{x} - 1)(\cos{x} - 1) = 0$

$\Leftrightarrow (\sin{x} - 1)(2\cos^2{x} + \cos{x} - 1) = 0$

$\Leftrightarrow (\sin{x} - 1)(\cos{x} + 1)(2\cos{x} - 1) = 0$

$\Leftrightarrow \left[\begin{array}{l} \sin{x} = 1\\\cos{x} = -1\\\cos{x} = \dfrac{1}{2}\end{array}\right. \Rightarrow \left[\begin{array}{l} x = \dfrac{\pi}{2} + 2k\pi\\x = \pi + 2k\pi\\x = \dfrac{\pm \pi}{3} + 2k\pi\end{array}\right. \,\, (k \in Z)$



#333174 Giải PT:$7\cos^2{x}+37\sin^{28}{x}=37$

Đã gửi bởi Phạm Hữu Bảo Chung on 08-07-2012 - 15:37 trong Phương trình, Hệ phương trình Lượng giác

Giải phương trình: $7cos^{2}x+37sin^{28}x=37$

Giải

Phương trình trên tương đương:
$37(\sin^{28}x - 1) + 7(1 - \sin^2{x}) = 0 \,\, (2)$


Áp dụng HĐT:
$$a^{14} - 1 = (a - 1)(a^{13} + a^{12} +… + a^2 + a + 1)$$

với $a = \sin^{2}x$

Phương trình (2) tương đương:
$37(\sin^2{x} - 1)(\sin^{26}x + \sin^{24}x + ... + 1) + 7(1 - \sin^2{x}) = 0$

$\Leftrightarrow (\sin^2{x} - 1)[37(\sin^{26}x + \sin^{24}x + … + \sin^2{x}) + 30] = 0$

$\Leftrightarrow \sin^2{x} = 1 \Rightarrow x = \dfrac{\pi}{2} + k\pi \,\, (k \in Z)$



#332930 Giải bpt $$\sqrt{x^{4}+2x^{2}-1}-\sqrt{2x^{4}+x^{2}+1}...

Đã gửi bởi Phạm Hữu Bảo Chung on 07-07-2012 - 21:11 trong Phương trình - hệ phương trình - bất phương trình

Mình nhấn Copy and Paste nên lộn dầu. Sorry hen :)

2) Tìm $m$ để hệ bpt có nghiệm duy nhất:
$\left\{\begin{array}{1}x^{2}-2mx\leq 0 \\\left | x-1+m \right |\leq 2m\end{array}\right.$

Giải

Từ phương trình thứ hai của hệ, suy ra:
$$2m \geq 0 \Leftrightarrow m \geq 0$$


Hệ ban đầu tương đương:
$\left\{\begin{array}{1}x(x - 2m)\leq 0 \\-2m \leq x-1+m \leq 2m\end{array}\right.$

$\Leftrightarrow \left\{\begin{array}{1}0 \leq x \leq 2m \\-3m + 1 \leq x\leq m + 1\end{array}\right.$


Hệ có nghiệm duy nhất khi: $\left[\begin{array}{l} m + 1 = 0\\2m = -3m + 1\end{array}\right. \Leftrightarrow \left[\begin{array}{l} m = -1\\m = \dfrac{1}{5}\end{array}\right.$

Do $m \geq 0 \Rightarrow m = \dfrac{1}{5}$

Thử lại thấy đúng. Vậy, với $m = \dfrac{1}{5}$ thì hệ BPT có nghiệm duy nhất.



#332919 Giải bpt $$\sqrt{x^{4}+2x^{2}-1}-\sqrt{2x^{4}+x^{2}+1}...

Đã gửi bởi Phạm Hữu Bảo Chung on 07-07-2012 - 20:54 trong Phương trình - hệ phương trình - bất phương trình

1) Giải bất phương trình :

$\sqrt{x^{4}+2x^{2}-1}-\sqrt{2x^{4}+x^{2}+1}< \sqrt{x^{2}+x}$

Giải

Điều kiện: $\left\{\begin{array}{l}x^{4}+2x^{2}-1 \geq 0\\2x^{4}+x^{2}+1 \geq 0\\x^2 + x \geq 0\end{array}\right. \Leftrightarrow \left\{\begin{array}{l}\left[\begin{array}{l} x^2 \geq \sqrt{2} - 1\\x^2 \leq - \sqrt{2} - 1 \,\, (VL)\end{array}\right.\\\dfrac{7}{4}x^2 + (\dfrac{1}{2}x^2 + 1)^2 \geq 0\\\left[\begin{array}{l} x \geq 0\\x \leq -1\end{array}\right.\end{array}\right.$

$\Leftrightarrow \left\{\begin{array}{l}\left[\begin{array}{l} x \geq \sqrt{\sqrt{2} - 1}\\x \leq - \sqrt{\sqrt{2} - 1}\end{array}\right.\\\left[\begin{array}{l} x \geq 0\\x \leq -1\end{array}\right.\end{array}\right. \Leftrightarrow \left[\begin{array}{l} x \geq \sqrt{\sqrt{2} - 1}\\x \leq -1\end{array}\right.$

Bất phương trình ban đầu tương đương:
$\dfrac{(x^4 + 2x^2 - 1) - (2x^4 + x^2 + 1)}{\sqrt{x^{4}+2x^{2}-1}+\sqrt{2x^{4}+x^{2}+1}} < \sqrt{x^2 + x}$


$\Leftrightarrow \dfrac{- x^4 + x^2 - 2}{\sqrt{x^{4}+2x^{2}-1}+\sqrt{2x^{4}+x^{2}+1}} < \sqrt{x^2 + x}$

Do
$\left\{\begin{array}{l}x^4 - x^2 + 2 = (x^2 - \dfrac{1}{2})^2 + \dfrac{7}{4} > 0\\\sqrt{x^{4}+2x^{2}-1}+\sqrt{2x^{4}+x^{2}+1} > 0\end{array}\right.$

$\Rightarrow VT < 0 \leq VF$


Vậy, BPT nghiệm đúng với mọi x thuộc tập xác định, tức:
$$x \in (- \propto; -1] \cup [\sqrt{\sqrt{2} - 1}; + \propto)$$



#332819 $\begin{cases}2-\sqrt{x^2y^4+2xy^2-y^4+1}= 2( 3-\sqrt 2-x...

Đã gửi bởi Phạm Hữu Bảo Chung on 07-07-2012 - 14:07 trong Phương trình - hệ phương trình - bất phương trình

$ \begin{cases}2-\sqrt{{x}^{2}{y}^{4}+2x{y}^{2}-{y}^{4}+1}= 2( 3-\sqrt{2}-x){y}^{2}\\ \sqrt{x-{y}^{2}}+x = 3 \,\, (2)\end{cases} $

Giải

ĐK: $\left\{\begin{array}{l}{x}^{2}{y}^{4}+2x{y}^{2}-{y}^{4}+1 \geq 0\\x \geq y^2 \geq 0\end{array}\right.$

Phương trình thứ nhất của hệ tương đương:
$2-\sqrt{(xy^2 + 1)^2 - y^4}= 2( 3-\sqrt{2}){y}^{2} - 2xy^2$


$\Leftrightarrow \sqrt{(xy^2 + 1 - y^2)(xy^2 + 1 + y^2)} = 2(1 + xy^2) - 2(3 - \sqrt{2})y^2 \,\, (3)$

Đặt $\left\{\begin{array}{l}a = \sqrt{xy^2 - y^2 + 1} \geq 0\\b = \sqrt{xy^2 + y^2 + 1} \geq 1\end{array}\right. \Rightarrow \left\{\begin{array}{l}xy^2 + 1 = \dfrac{a^2 + b^2}{2}\\y^2 = \dfrac{b^2 - a^2}{2}\end{array}\right.$

Phương trình (3) trở thành:
$ab = 2.\dfrac{a^2 + b^2}{2} - 2(3 - \sqrt{2})\dfrac{b^2 - a^2}{2}$


$\Leftrightarrow ab = a^2 + b^2 + (3 - \sqrt{2})(a^2 - b^2)$

$\Leftrightarrow (4 - \sqrt{2})a^2 - ab - (2 - \sqrt{2})b^2 = 0 $

$\Leftrightarrow (4 - \sqrt{2})(\dfrac{a}{b})^2 - \dfrac{a}{b} - 2 + \sqrt{2} = 0$


$\Rightarrow \left[\begin{array}{l} \dfrac{a}{b} = \dfrac{1}{\sqrt{2}}\\\dfrac{a}{b} = \dfrac{-\sqrt{2}}{3 + \sqrt{2}}\end{array}\right.$

Do $a \geq 0, b \geq 1 \Rightarrow \dfrac{a}{b} = \dfrac{1}{\sqrt{2}}$

$\Leftrightarrow \dfrac{xy^2 - y^2 + 1}{xy^2 + y^2 + 1} = \dfrac{1}{2} \Leftrightarrow xy^2 - 3y^2 + 1 = 0 \,\, (4)$


Dễ thấy $x \neq 3$ (vì x = 3 thì (4) tương đương với 0 = 1. Vô lý)

Từ (4), suy ra: $y^2 = \dfrac{-1}{x - 3}$

Thế vào (2), ta được:
$\sqrt{x+ \dfrac{1}{x - 3}}= 3 - x$


$\Leftrightarrow \left\{\begin{array}{l}x \leq 3\\x^3 - 10x^2 + 30x - 28 = 0\end{array}\right. \Leftrightarrow \left\{\begin{array}{l}x \leq 3\\(x - 2)(x^2 - 8x + 14) = 0\end{array}\right.$

$\Leftrightarrow \left\{\begin{array}{l}\left[\begin{array}{l} x = 2\\x = 4 \pm \sqrt{2}\end{array}\right.\\x \leq 3\end{array}\right. \Leftrightarrow \left[\begin{array}{l} x = 2\\x = 4 - \sqrt{2}\end{array}\right.$


- Với x = 2, suy ra $y = \pm -1$

- Với $x = 4 - \sqrt{2} \Rightarrow y = \pm \sqrt{\sqrt{2} + 1}$



#332744 GHPT $2x_{1}=x_{2}+\frac{1}{x_{2}}$.......

Đã gửi bởi Phạm Hữu Bảo Chung on 07-07-2012 - 06:29 trong Phương trình, hệ phương trình và bất phương trình

Giải

ĐK: $x_1, x_2, x_3..., x_{2000} \neq 0$
Từ phương trình (1), ta thấy:
$|2x_1| = |x_2 + \dfrac{1}{x_2}| = |x_2| + |\dfrac{1}{x_2}| \geq 2$

$\Rightarrow \left[\begin{array}{l} x_1 \leq - 1\\x_1 \geq 1\end{array}\right.$


Hoàn toàn tương tự, ta sẽ chứng minh được:
$$x_2, x_3, x_4…., x_{2000} \in (- \propto; - 1] \cup [1; \propto)$$


Mặt khác, từ các phương trình của hệ, dễ dàng nhận thấy: Các giá trị $x_1, x_2, x_3..., x_{2000}$ luôn cùng dấu. Do đó, tổng hợp những điều nói trên, ta sẽ có điều kiện của các ẩn số là:
$\left[\begin{array}{l} \left\{\begin{array}{l}x_1 \leq - 1\\x_2 \leq -1\\...\\x_{2000} \leq -1\end{array}\right.\\\left\{\begin{array}{l}x_1 \geq 1\\x_2 \geq 1\\...\\x_{2000} \geq 1\end{array}\right.\end{array}\right. \,\, (A)$

Không mất tính tổng quát, giả sử: $x_1 = max(x_1; x_2…; x_{2000})$

$\Rightarrow x_1 \geq x_{2000}$

Khi đó, từ phương trình (1) và phương trình (2000), suy ra:
$x_2 + \dfrac{1}{x_2} \geq x_1 + \dfrac{1}{x_1} \Leftrightarrow (x_2 - x_1) + \dfrac{x_1 - x_2}{x_1.x_2} \geq 0$

$\Leftrightarrow (x_2 - x_1)(1 - \dfrac{1}{x_1.x_2}) \geq 0 \,\, (B)$


Từ (A), ta có:
$\left[\begin{array}{l} \left\{\begin{array}{l}x_1 \geq 1\\x_2 \geq 1\end{array}\right.\\\left\{\begin{array}{l}x_1 \leq - 1\\x_2 \leq -1\end{array}\right.\end{array}\right. \Rightarrow x_1.x_2 \geq 1$

$\Rightarrow \dfrac{1}{x_1.x_2} \leq 1 \Leftrightarrow 1 - \dfrac{1}{x_1.x_2} \geq 0$


* Nếu $x_1.x_2 = 1$ thì $\left[\begin{array}{l} \left\{\begin{array}{l}x_1 = 1\\x_2 = 1\end{array}\right.\\\left\{\begin{array}{l}x_1 = -1\\x_2\end{array}\right.\end{array}\right.$

- Với $x_1 = x_2 = 1 \Rightarrow x_3 = x_4 = ….= x_{2000} = 1$

- Với $x_1 = x_2 = - 1 \Rightarrow x_3 = x_4 = ….= x_{2000} = -1$



* Nếu $x_1.x_2 > 1$
Do đó, (B) tương đương: $x_2 \geq x_1 = max(x_1; x_2;…; x_{2000})$. Khi đó: $x_2 = max(x_1; x_2;…; x_{2000})$.
Suy ra: $x_1 = x_2$

Từ các phương trình trong hệ và điều kiện $x_1 = x_2$, ta suy ra được:
$$x_1 = x_2 = x_3 =….= x_{2000}$$


Khi đó, hệ ban đầu tương đương:
$\left\{\begin{array}{l}x_1 = x_2 =… = x_{2000}\\2x_1 = x_1 + \dfrac{1}{x_1}\end{array}\right. \Leftrightarrow \left[\begin{array}{l} x_1 = x_2 =… = x_{2000} = 1\\x_1 = x_2 =… = x_{2000} = -1\end{array}\right.$


Do $x_1.x_2 > 1$ nên trong TH này, hệ vô nghiệm.

KẾT LUẬN: Hệ có nghiệm:

$\left[\begin{array}{l} x_1 = x_2 =… = x_{2000} = 1\\x_1 = x_2 =… = x_{2000} = -1\end{array}\right.$



#330265 Đề thi tuyển sinh chuyên Thăng Long Đà Lạt 2012

Đã gửi bởi Phạm Hữu Bảo Chung on 29-06-2012 - 16:29 trong Tài liệu - Đề thi

Không hiểu sao khi mình nhấn sửa ở bài viết phía trên thì mấy dấu $\geq $ đều trở thành #6... gì vậy nhỉ. Thông cúm cho tớ nghen!
Câu 9.(1,5) Chứng minh rằng không tồn tại các số tự nhiên m, n sao cho
$\frac{1}{4}(m-n)(m+n)[1+(-1)^{m+n}]=2013$

@minhtuyb: Với m + n chẵn, ta đâu thể khẳng định được rằng: $1 + (- 1)^{m + n} = 0$


Giải

Ta có thể đánh giá như sau:

Dễ thấy: $m > n \geq 0$ vì nếu $m \leq n$ thì $VT \leq 0 \neq VF$


- Với m, n khác tính chẵn, lẻ. Khi đó m + n lẻ, suy ra:
$$1 + (-1)^{m + n} = 1 - 1 = 0$$
Khi đó, $VT = 0 \neq 2013 = VF$.

- Với m, n cùng tính chẵn lẻ. Khi đó:
$m + n; m - n \, \vdots \, 2$ và $1 + (- 1)^{m + n} = 2$

Suy ra: $\frac{1}{4}(m-n)(m+n)[1+(-1)^{m+n}] \, \vdots \, 2 $
Mà: $2003 \not \vdots \, 2$
Vì thế, $VT \neq VF$

Nói tóm lại, không tồn tại các số tự nhiên m, n thỏa mãn đề bài.



#330259 Đề thi tuyển sinh chuyên Thăng Long Đà Lạt 2012

Đã gửi bởi Phạm Hữu Bảo Chung on 29-06-2012 - 16:15 trong Tài liệu - Đề thi

Câu 8.(1,5) Cho x,y là hai số dương thỏa: $x^3+y^3=x-y$.CM $x^2+y^2<1$

Giải

Do x, y dương nên $x^3 + y^3 > 0 \Rightarrow x > y$

Ta có: $x^3+y^3=x-y \Rightarrow \dfrac{x^3 + y^3}{x - y} = 1$

Do đó, để CM $x^2+y^2<1$, ta chỉ cần CM $x^2 + y^2 < \dfrac{x^3 + y^3}{x - y} \,\, (1)$

Ta thấy:
$(1) \Leftrightarrow (x^2 + y^2)(x - y) < x^3 + y^3$

$\Leftrightarrow x^3 - x^2y + y^2x - y^3 < x^3 + y^3$

$\Leftrightarrow 2y^3 + x^2y - y^2x > 0 \Leftrightarrow y(2y^2 - xy + x^2) > 0$


$\Leftrightarrow y[(x - \dfrac{y}{2})^2 + \dfrac{7y^2}{4}] > 0 $

BĐT trên luôn đúng với x, y > 0.



#330253 \[\sum {\left| {\frac{{a - b}}{{a + b}}} \right|...

Đã gửi bởi Phạm Hữu Bảo Chung on 29-06-2012 - 15:59 trong Bất đẳng thức và cực trị

Với tam giác vuông ABC có độ dài 3 cạnh lần lượt là a = 3, b = 4, c = 5.
Ta thấy: $VT > \left |\dfrac{c - a}{c + a} \right| = \left |\dfrac{2}{8} \right| = \dfrac{1}{4} > \dfrac{1}{8}$


:| ?!



#330105 Giải phương trình $2x^2+3\sqrt[3]{x^3-9}=\frac{10}{x}$

Đã gửi bởi Phạm Hữu Bảo Chung on 28-06-2012 - 22:57 trong Phương trình, hệ phương trình và bất phương trình

Giải phương trình: $2{x^2} + 3\sqrt[3]{{{x^3} - 9}} = \frac{{10}}{x}$

@donghaidhtt: :) Chắc cậu nhầm 1 chút:
Với TH x < 0, VT vẫn có thể nhỏ hơn 0. Đơn cử như với x = -1.

Cậu chỉ mới CM được rằng: $2x^2 > 3\sqrt[3]{{{x^3} - 9}}$ chứ không phải là $2x^2 > - 3\sqrt[3]{{{x^3} - 9}}$
Vì vậy, TH x < 0 mình nghĩ chưa đúng. (Hồi nãy đến giờ mình cũng tìm cách CM nhưng không khả quan :()


Giải

ĐK: $x \neq 0$
Phương trình ban đầu tương đương:
$2x^3 + 3x\sqrt[3]{x^3 - 9} = 10$

$\Leftrightarrow 2x^3 + 3\sqrt[3]{x^6 - 9x^3} = 10$

Đặt $x^3 = a $, suy ra:
$2a + 3\sqrt[3]{a^2 - 9a} = 10 \Leftrightarrow 3\sqrt[3]{a^2 - 9a} = 10 - 2a$

$\Leftrightarrow 27(a^2 - 9a) = 8(125 - 75a + 15a^2 - a^3) $

$\Leftrightarrow 8a^3 - 93a + 357a - 1000 = 0 \Leftrightarrow (a - 8)(8a^2 - 29a + 125) = 0$

$\Leftrightarrow a = 8 $ (Do phương trình $8a^2 - 29a + 125 = 0$ vô nghiệm)


$\Rightarrow x^3 = 2 \Leftrightarrow x = 2 ™$

P/S: Hơi thực dụng nhưng khá hiệu quả nhỉ? :)



#329926 Giải HPT:\[\left\{\begin{array}{l} {x^2}+2xy+y=0\...

Đã gửi bởi Phạm Hữu Bảo Chung on 28-06-2012 - 13:14 trong Phương trình - hệ phương trình - bất phương trình

Giải hệ phương trình:
\[\left\{\begin{array}{l} {x^2}+2xy+y=0\\ {x^3}+3xy+2\sqrt {y+1} \left( {x+\sqrt {{x^2}y + 2} } \right) = 4 \end{array}\right.\]


Giải

ĐK:
$\left\{\begin{array}{l}y + 1 \geq 0\\xy^2 + 2 \geq 0 \end{array}\right. \Leftrightarrow \left\{\begin{array}{l}y \geq -1\\xy^2 + 2 \geq 0\end{array}\right.$

Phương trình thứ hai của hệ tương đương:
$x(x^2 + y) + 2xy + 2\sqrt{(y + 1)}( x+\sqrt {x^2y + 2}) = 4$

$\Rightarrow x.(-2xy) - (x^2 + y) + 2\sqrt{(y + 1)}( x+\sqrt {x^2y + 2}) - 4 = 0$


$\Leftrightarrow 2x^2y+ x^2 + y - 2.x.\sqrt{y + 1} - 2\sqrt{(y + 1)(x^2y +2)} + 4 = 0$

$\Leftrightarrow [x^2(y + 1) - 2.x.\sqrt{y + 1} + 1] + [x^2y + 2 - 2\sqrt{(y + 1)(x^2y +2)} + y + 1] = 0$


$\Leftrightarrow (x\sqrt{y + 1} - 1)^2 + (\sqrt{x^2y + 2} - \sqrt{y + 1})^2 = 0$

$\Rightarrow \left\{\begin{array}{l}x\sqrt{y + 1} - 1 = 0\\\sqrt{x^2y + 2} - \sqrt{y + 1} = 0\end{array}\right. \Leftrightarrow \left\{\begin{array}{l}x\sqrt{y + 1} = 1\\\sqrt{x^2y + 2} = \sqrt{y + 1}\end{array}\right.$


$\Leftrightarrow \left\{\begin{array}{l}x \geq 0\\x^2(y + 1) = 1 \,\, (1)\\x^2y - y = -1 \,\, (2)\end{array}\right.$

Lấy (1) - (2) vế theo vế, ta được:
$x^2 + y = 2 \Rightarrow y = 2 - x^2 \leq 2$


Thế $y = 2 - x^2$ vào (1), ta có phương trình:
$x^2(3 - x^2) = 1 \Leftrightarrow x^4 - 3x^2 + 1 = 0$


$\Leftrightarrow \left[\begin{array}{l} x = \pm \sqrt{\dfrac{3 + \sqrt{5}}{2}}\\x = \pm \sqrt{\dfrac{3 - \sqrt{5}}{2}}\end{array}\right.$

Do $x \geq 0$ nên: $\left[\begin{array}{l} x = \sqrt{\dfrac{3 + \sqrt{5}}{2}}\\x = \sqrt{\dfrac{3 - \sqrt{5}}{2}}\end{array}\right.$

- Với $x = \sqrt{\dfrac{3 + \sqrt{5}}{2}} \Rightarrow y = \dfrac{1 - \sqrt{5}}{2} < 2$
- Với $x = \sqrt{\dfrac{3 - \sqrt{5}}{2}} \Rightarrow y = \dfrac{1 + \sqrt{5}}{2} < 2$

Thử lại 2 cặp giá trị nói trên, ta chọn cặp $(x; y) = (\sqrt{\dfrac{3 + \sqrt{5}}{2}}; \dfrac{1 - \sqrt{5}}{2})$



#329872 GHPT: 1.$\left\{\begin{matrix} x^{3}y-y^{4}=28 &...

Đã gửi bởi Phạm Hữu Bảo Chung on 28-06-2012 - 08:32 trong Phương trình - hệ phương trình - bất phương trình

6. $\left\{\begin{matrix} x+y+z=6 & \\ x^{2}+y^{2}+z^{2}=18 & \\ \sqrt{x}+\sqrt{y}+\sqrt{z}=4 & \end{matrix}\right.$

Giải

ĐK: $x, y, z \geq 0$
Ta có:
$\sqrt{x}+\sqrt{y}+\sqrt{z}=4 \Leftrightarrow (\sqrt{x}+\sqrt{y}+\sqrt{z})^2 = 16$

$\Leftrightarrow x + y + z + 2(\sqrt{xy} + \sqrt{yz} + \sqrt{zx}) = 16$

$\Rightarrow \sqrt{xy} + \sqrt{yz} + \sqrt{zx} = 5 \,\, (1)$


Ta lại có:
$x + y + z = 6 \Leftrightarrow (x + y + z)^2 = 36$

$\Leftrightarrow x^2 + y^2 + z^2 + 2(xy + yz + zx) = 36$

$\Rightarrow xy + yz + zx = 9 $


Bình phương 2 vế của (1), ta được:
$(\sqrt{xy} + \sqrt{yz} + \sqrt{zx})^2 = 25$

$\Leftrightarrow xy + yz + zx + 2\sqrt{xyz}(\sqrt{x} + \sqrt{y} + \sqrt{z}) = 25$

$\Rightarrow 9 + 2.\sqrt{xyz}.4 = 25 \Leftrightarrow xyz = 4 \,\, (2)$

Dễ thấy, $x, y, z \neq 0$. Với ĐK này, từ (2) suy ra:
$$xy = \dfrac{4}{z}$$


Ta thấy: $x + y + z = 6 \Leftrightarrow x + y = 6 - z$

$\Leftrightarrow \left\{\begin{array}{l}z \leq 6\\x^2 + y^2 + 2xy = 36 - 12z + z^2\end{array}\right. \Rightarrow \left\{\begin{array}{l}z \leq 6\\18 - z^2 + 2.\dfrac{4}{z} = z^2 - 12z + 36\end{array}\right.$

$\Leftrightarrow \left\{\begin{array}{l}z \leq 6\\z^3 - 6z^2 + 9z - 4 = 0\end{array}\right. \Leftrightarrow \left\{\begin{array}{l}z \leq 6\\(z - 1)^2(z - 4) = 0\end{array}\right.$

$\Leftrightarrow \left[\begin{array}{l} z = 1\\z = 4\end{array}\right.$


Hoàn toàn tương tự, ta cũng có:
$\left\{\begin{array}{l}\left[\begin{array}{l} x = 1\\x = 4\end{array}\right.\\\left[\begin{array}{l} y = 1\\y = 4\end{array}\right.\end{array}\right.$


Lần lượt lựa chọn các nghiệm của PT. Nếu 1 ẩn có giá trị bằng 4 thì các ẩn còn lại có giá trị là 1. Suy ra, hệ có các nghiệm:
$(x; y; z) = (4; 1; 1); (1; 4; 1) = (1; 1; 4)$



#329816 $\sqrt[3]{3x^{2}-3x+3}-\sqrt{\frac{x^{3}}{3}-\frac{3...

Đã gửi bởi Phạm Hữu Bảo Chung on 27-06-2012 - 22:19 trong Phương trình - hệ phương trình - bất phương trình

$\sqrt[3]{3x^{2}-3x+3}-\sqrt{\frac{x^{3}}{3}-\frac{3}{4}}=\frac{1}{2}$
Bài này cũng có thể giải theo cách này:

Giải

ĐK: $x \geq \sqrt[3]{\dfrac{9}{4}}$
Đặt $\left\{\begin{array}{l}a = \sqrt[3]{3x^{2}-3x+3} = \sqrt[3]{3(x - \dfrac{1}{2})^2 + \dfrac{9}{4}} \geq \sqrt[3]{\dfrac{9}{4}}\\b = \sqrt{\frac{x^{3}}{3}-\frac{3}{4}} \geq 0\end{array}\right.$

Theo đề bài, ta có: $a - b = \dfrac{1}{2} \Rightarrow b = a - \dfrac{1}{2} \,\, (1)$

Mặt khác, ta thấy:
$a^3 + 3b^2 = 3x^2 - 3x + 3 + x^3 - \dfrac{9}{4} = x^3 + 3x^2 - 3x + \dfrac{3}{4} \,\, (2)$

Thế (1) vào (2), ta có:
$a^3 + 3(a - \dfrac{1}{2})^2 = x^3 + 3x^2 - 3x + \dfrac{3}{4}$

$\Leftrightarrow a^3 + 3a^2 - 3a + \dfrac{3}{4} = x^3 + 3x^2 - 3x + \dfrac{3}{4}$

$\Leftrightarrow (a - x)(a^2 + ax + x^2 + 3a + 3x - 3) = 0$


Dễ thấy: $a^2 + ax + x^2 + 3(a + x) - 3 = (a + \dfrac{x}{2})^2 + \dfrac{3x^2}{4} + 3(a + x) - 3 > 0 \forall a; x \geq \sqrt[3]{\dfrac{9}{4}} $

Do đó: $a = x \Rightarrow \sqrt[3]{3x^2 - 3x + 3} = x$

$\Leftrightarrow x^3 - 3x^2 + 3x - 1 = 2 \Leftrightarrow (x - 1)^3 = 2$


$\Leftrightarrow x = 1 + \sqrt[3]{2}$



#329361 Topic phương trình, hệ phương trình vô tỉ

Đã gửi bởi Phạm Hữu Bảo Chung on 26-06-2012 - 16:21 trong Phương trình, hệ phương trình và bất phương trình

@luxubuhl: Phương trình có 1 nghiệm gần bằng 1, 027 nữa bạn à!



#329348 Giải phương trình: $\Sigma\frac{x+\sqrt{3}}{\sqrt{x}...

Đã gửi bởi Phạm Hữu Bảo Chung on 26-06-2012 - 15:23 trong Phương trình - hệ phương trình - bất phương trình

Giải phương trình: $$\dfrac{x+\sqrt{3}}{\sqrt{x}+\sqrt{x+\sqrt{3}}}+ \dfrac{x-\sqrt{3}}{\sqrt{x}-\sqrt{x-\sqrt{3}}}=\sqrt{x}$$

Giải

ĐK: $x \geq \sqrt{3}$

Phương trình ban đầu tương đương:
$\dfrac{(x + \sqrt{3})(\sqrt{x+\sqrt{3}} - \sqrt{x})}{x + \sqrt{3} - x} + \dfrac{(x-\sqrt{3})(\sqrt{x}+\sqrt{x-\sqrt{3}})}{x - (x - \sqrt{3})} = \sqrt{x}$


$\Leftrightarrow \dfrac{(x + \sqrt{3})\sqrt{x + \sqrt{3}} + (x - \sqrt{3})\sqrt{x - \sqrt{3}} - 2\sqrt{3x}}{\sqrt{3}} = \sqrt{x}$

$\Leftrightarrow (x + \sqrt{3})\sqrt{x + \sqrt{3}} + (x - \sqrt{3})\sqrt{x - \sqrt{3}} = 3\sqrt{3x} \,\, (2)$


Đặt $\left\{\begin{array}{l}a = \sqrt{x + \sqrt{3}} > 0\\b = \sqrt{x - \sqrt{3}} \geq 0\end{array}\right. \,\, (a > b)$

$\Rightarrow \left\{\begin{array}{l}a^2 + b^2 = 2x\\a^2 - b^2 = 2\sqrt{3}\end{array}\right.$


Phương trình (2) trở thành:
$a^3 + b^3 = 3.\dfrac{a^2 - b^2}{2}.\sqrt{\dfrac{a^2 + b^2}{2}}$

$\Leftrightarrow (a + b)(a^2 - ab + b^2 - \dfrac{3}{2}(a - b)\sqrt{\dfrac{a^2 + b^2}{2}}) = 0$

$\Leftrightarrow 2(a^2 - ab + b^2) - 3(a - b)\sqrt{\dfrac{a^2 + b^2}{2}} = 0$


(Do $a > 0; b \geq 0 \Rightarrow a + b > 0$)

$\Leftrightarrow (a - b)^2 + (a^2 + b^2) - 3(a - b)\sqrt{\dfrac{a^2 + b^2}{2}} = 0$

$\Leftrightarrow \sqrt{2}(a - b)^2 - 3(a - b)\sqrt{a^2 + b^2} + \sqrt{2}(a^2 + b^2) = 0$

$\Leftrightarrow \sqrt{2}\dfrac{(a - b)^2}{a^2 + b^2} - 3\dfrac{a - b}{\sqrt{a^2 + b^2}} + \sqrt{2} = 0 \,\, (a^2 + b^2 > 0)$

$\Leftrightarrow \left[\begin{array}{l} \dfrac{a - b}{\sqrt{a^2 + b^2}} = \sqrt{2}\\\dfrac{a - b}{\sqrt{a^2 + b^2}} = \dfrac{1}{\sqrt{2}}\end{array}\right.$


Dễ thấy $\dfrac{a - b}{\sqrt{a^2 + b^2}} \leq 1 \forall \, a > 0; b \geq 0; a > b$

$\Rightarrow \dfrac{a - b}{\sqrt{a^2 + b^2}} = \dfrac{1}{\sqrt{2}}$

$\Leftrightarrow 2(a^2 - 2ab + b^2) = a^2 + b^2$

$\Leftrightarrow a^2 - 4ab + b^2 = 0 \Leftrightarrow \left[\begin{array}{l} a = (2 + \sqrt{3})b\\a = (2 - \sqrt{3})b\end{array}\right.$


$\Leftrightarrow \left[\begin{array}{l} a^2 = (7 + 4\sqrt{3})b^2\\a^2 = (7 - 4\sqrt{3})b^2\end{array}\right. \Rightarrow \left[\begin{array}{l} x + \sqrt{3} = (7 + 4\sqrt{3})(x - \sqrt{3})\\x + \sqrt{3} = (7 - 4\sqrt{3})(x - \sqrt{3})\end{array}\right.$

$\Leftrightarrow \left[\begin{array}{l} x = 2\\x = - 2\end{array}\right.$

Kết hợp với ĐK, suy ra x = 2.


P/S: Trông thật lằng nhằng!



#329293 ĐỀ TUYỂN SINH (ko chuyên) thpt CHUYÊN HÀ TĨNH 2012-2013

Đã gửi bởi Phạm Hữu Bảo Chung on 26-06-2012 - 10:51 trong Tài liệu - Đề thi

3b)Giải phương trình $\sqrt{x+1} + x + 3 = \sqrt{1-x}+3\sqrt{1-x^{2}}$

Giải

ĐK: $- 1\leq x \leq 1$
Đặt $\left\{\begin{array}{l}a = \sqrt{x + 1} \geq 0\\b = \sqrt{1 - x} \geq 0\end{array}\right. \Rightarrow \left\{\begin{array}{l}x = \dfrac{a^2 - b^2}{2}\\1 = \dfrac{a^2 + b^2}{2}\end{array}\right.$

$\Rightarrow x + 3 = \dfrac{a^2 - b^2}{2} + \dfrac{3(a^2 + b^2)}{2} = 2a^2 + b^2$


Phương trình ban đầu trở thành:
$a + 2a^2 + b^2 = b + 3ab \Leftrightarrow (2a^2 - 3ab + b^2) + a - b = 0$

$\Leftrightarrow (a - b)(2a - b + 1) = 0 \Leftrightarrow \left[\begin{array}{l} a = b\\b = 2a + 1\end{array}\right.$

$\Rightarrow \left[\begin{array}{l} \sqrt{x + 1} = \sqrt{1 - x}\\\sqrt{1 - x} = 2\sqrt{x + 1} + 1\end{array}\right. \Leftrightarrow \left[\begin{array}{l} x = 0\\1 - x = 4(x + 1) + 1 + 4\sqrt{x + 1}\end{array}\right.$

$\Leftrightarrow \left[\begin{array}{l} x = 0\\-5x - 4 = 4\sqrt{x + 1}\end{array}\right. \Leftrightarrow \left[\begin{array}{l} x = 0\\\left\{\begin{array}{l}-5x - 4 \geq 0\\25x^2 + 40x + 16 = 16(x + 1)\end{array}\right.\end{array}\right.$


$\Leftrightarrow \left[\begin{array}{l} x = 0\\\left\{\begin{array}{l}x \leq \dfrac{- 4}{5}\\x(25x + 24) = 0\end{array}\right.\end{array}\right.$

$\Leftrightarrow \left[\begin{array}{l} x = 0\\\left\{\begin{array}{l}x \leq \dfrac{-4}{5}\\\left[\begin{array}{l} x = \dfrac{-24}{25}\\x = 0\end{array}\right.\end{array}\right.\end{array}\right. \Leftrightarrow \left[\begin{array}{l} x = 0\\x = \dfrac{-24}{25}\end{array}\right.$



#329274 Tìm điều kiện của $m$ để PT có 4 nghiệm $x_1,x_2,x_3,x_4...

Đã gửi bởi Phạm Hữu Bảo Chung on 26-06-2012 - 09:25 trong Đại số

Cho PT x4 +(2m2 - 1)x2 + 7m - 1 = 0 (1)
tìm điều kiện của m để PT có 4 nghiệm x1,x2,x3,x4 phân biệt và x12 + x22 + x32 + x42 =10


Giải

Đặt $t = x^2 \geq 0$, phương trình (1) trở thành:
$$t^2 + (2m^2 - 1).t + 7m - 1 = 0 \,\, (2)$$


Phương trình ban đầu có 4 nghiệm phân biệt khi và chỉ khi phương trình (2) có 2 nghiệm phân biệt dương. Điều này đồng nghĩa với:
$\left\{\begin{array}{l}\Delta_{(2)} = (2m^2 - 1)^2 - 4(7m - 1) > 0\\S_{(2)} = t_1 + t_2 > 0\\P_{(2)} = t_1.t_2 > 0\end{array}\right. \Rightarrow \left\{\begin{array}{l}4m^4 - 4m^2 - 28m + 5 > 0\\1 - 2m^2 > 0\\7m - 1 > 0\end{array}\right.$


$\Leftrightarrow \left\{\begin{array}{l}4m^4 - 4m^2 - 28m + 5 > 0\\\dfrac{- 1}{\sqrt{2}} < m < \dfrac{1}{\sqrt{2}}\\m < \dfrac{1}{7}\end{array}\right. \Leftrightarrow \left\{\begin{array}{l}4m^4 - 4m^2 - 28m + 5 > 0\\\dfrac{- 1}{\sqrt{2}} < m < \dfrac{1}{7}\end{array}\right.$

Ta sẽ xét tới điều kiện:
$$x_1^2 + x_2^2 + x_3^2 + x_4^2 = 10 \,\, (3)$$

Gọi $t_1; t_1$ lần lượt là 2 nghiệm dương của phương trình (2)

$\Rightarrow \left[\begin{array}{l} x^2 = t_1\\x^2 = t_2\end{array}\right. \Leftrightarrow \left[\begin{array}{l} x = \pm \sqrt{t_1}\\x = \pm \sqrt{t_2}\end{array}\right.$


Không mất tính tổng quát, giả sử $x_1 = \sqrt{t_1}; x_2 = - \sqrt{t_1}; x_3 = \sqrt{t_2}; x_4 = -\sqrt{t_2}$

$\Rightarrow x_1^2 + x_2^2 + x_3^2 + x_4^2 = 2t_1 + 2t_2 = 2(1 - 2m^2) \,\, (4)$


Từ (3) và (4), suy ra:
$2(1 - 2m^2) = 10 \Leftrightarrow 1 - 2m^2 = 5 \Leftrightarrow m^2 = -2$


Liệu có nhầm lẫn gì ở đây không nhỉ? :3



#329263 Giải hệ phương trình:\[\left\{ \begin{array}{l} \sqr...

Đã gửi bởi Phạm Hữu Bảo Chung on 26-06-2012 - 08:52 trong Phương trình - hệ phương trình - bất phương trình

Giải hệ phương trình:
$\left\{\begin{array}{l}\sqrt{2x + y} = 2 - 2x - y\\\sqrt{x + 2} + \sqrt{2x - y + 2} = 2x + 1\end{array}\right.$

Giải

ĐK:
$\left\{\begin{array}{l}2x + y \geq 0\\x \geq -2\\2x - y + 2 \geq 0\end{array}\right. \Leftrightarrow \left\{\begin{array}{l}-2x \leq y \leq 2x + 2\\x \geq -2\end{array}\right.$

$\Rightarrow \left\{\begin{array}{l}x \geq \dfrac{-1}{2}\\-2x \leq y \leq 2x + 2\end{array}\right.$


Phương trình thứ nhất của hệ tương đương:
$2x + y + \sqrt{2x + y} - 2 = 0$

$\Leftrightarrow (\sqrt{2x + y} - 1)(\sqrt{2x + y} + 2) = 0$

$\Leftrightarrow \left[\begin{array}{l} \sqrt{2x + y} = 1\\\sqrt{2x + y} = -2\end{array}\right.$

Do $\sqrt{2x + y} \geq 0 \Rightarrow \sqrt{2x + y} = 1 \Leftrightarrow 2x + y = 1$

$$\Leftrightarrow y = 1 - 2x$$

Khi đó, phương trình thứ 2 của hệ trở thành:
$\sqrt{x + 2} + \sqrt{2x - (1 - 2x) + 2} = 2x + 1$

$\Leftrightarrow \sqrt{x + 2} + \sqrt{4x + 1} = 2x + 1 \,\,\, (x \geq \dfrac{-1}{4})$


$\Leftrightarrow \sqrt{x + 2} - 2 + \sqrt{4x + 1} - 3 - 2x + 4 = 0$

$\Leftrightarrow \dfrac{x - 2}{\sqrt{x + 2} + 2} + \dfrac{4x - 8}{\sqrt{4x + 1} + 3} -2(x - 2) = 0$

$\Leftrightarrow (x - 2)(\dfrac{1}{\sqrt{x + 2} + 2} + \dfrac{4}{\sqrt{4x + 1} + 3} - 2) = 0$

$\Leftrightarrow \left[\begin{array}{l} x = 2\\\dfrac{1}{\sqrt{x + 2} + 2} + \dfrac{4}{\sqrt{4x + 1} + 3} = 2\end{array}\right.$


- Với x = 2, suy ra y = -3
Hệ có nghiệm $(x; y) = (2; -3)$
- Với $\dfrac{1}{\sqrt{x + 2} + 2} + \dfrac{4}{\sqrt{4x + 1} + 3} = 2 $
Nhận thấy:
$VT \leq \dfrac{1}{2} + \dfrac{4}{3} = \dfrac{11}{6} < 2 = VF$

Do đó PT nói trên vô nghiệm. Khi đó, hệ vô nghiệm.

Kết luận: Hệ có nghiệm duy nhất; $(x; y) = (2; -3)$